Computing area of equilateral triangle on a sphere

Click For Summary
The discussion focuses on calculating the area of an equilateral triangle on a sphere with radius R = 1, using the formula A = α²R². The user initially assumes each angle of the triangle is 90 degrees, leading to an area calculation of A = π/2. Concerns are raised about the choice of angle, as selecting different angles could yield different area results. The inclusion-exclusion principle is mentioned as necessary for a more general solution applicable to any angle. The user is advised to explore visual resources to better understand the problem rather than relying solely on their current assumptions.
tainted
Messages
28
Reaction score
0

Homework Statement



Suppose T is an equilateral triangle on the sphere of radius R = 1. Let \alpha denote the angle at any of the three vertices’s of the triangle. (Recall that 3\alpha > n.) Use the result of the last problem on the previous homework and the inclusion - exclusion principle (together with an orange and a knife) to compute the area of T .

Homework Equations



The result to the last problem on the previous homework is A = \alpha2R^2

The Attempt at a Solution


I assumed that all angle on the equilateral triangle where 90 degrees or \frac{\pi}{2}; therefore making the volume equal to 1/8 that of the whole sphere

So I did
A = \alpha2R^2 where A is the area of T
A = \frac{\pi}{2}2R^2
A = \pi*R^2 That would be the area of 1/4 of the sphere overall, but because I am taking the area of an equilateral triangle, I took half of that to get
A = \frac{\pi}{2}R^2
A = \frac{\pi}{2}*1
A = \frac{\pi}{2}

Would that be correct? I just kind of picked 90 degrees or \frac{\pi}{2} for \alpha, but I assume it could be anything between 60 and up to 90 degrees which would change my answer. How do I know which angle to pick?
 
Last edited:
Physics news on Phys.org
I am concerned that my answer will be completely different if I used an angle other than 90 degrees. Also I do not know what an orange and knife have to do with this problem.
 
You are supposed to do it for ANY angle using inclusion/exclusion. Look at the pictures here: http://planetmath.org/AreaOfASphericalTriangle.html Just look at the pictures, ignore the solution and try to work it out for yourself.
 
Last edited by a moderator:
Question: A clock's minute hand has length 4 and its hour hand has length 3. What is the distance between the tips at the moment when it is increasing most rapidly?(Putnam Exam Question) Answer: Making assumption that both the hands moves at constant angular velocities, the answer is ## \sqrt{7} .## But don't you think this assumption is somewhat doubtful and wrong?

Similar threads

  • · Replies 7 ·
Replies
7
Views
1K
  • · Replies 3 ·
Replies
3
Views
3K
  • · Replies 1 ·
Replies
1
Views
1K
Replies
4
Views
1K
Replies
3
Views
2K
Replies
5
Views
3K
  • · Replies 2 ·
Replies
2
Views
1K
  • · Replies 16 ·
Replies
16
Views
2K
Replies
14
Views
2K
  • · Replies 5 ·
Replies
5
Views
1K